LSAT and Law School Admissions Forum

Get expert LSAT preparation and law school admissions advice from PowerScore Test Preparation.

User avatar
 Dave Killoran
PowerScore Staff
  • PowerScore Staff
  • Posts: 5850
  • Joined: Mar 25, 2011
|
#45765
Complete Question Explanation
(The complete setup for this game can be found here: lsat/viewtopic.php?t=8976)

The correct answer choice is (C)

This is a question that separates students. Some students recognize the importance of the doubled product during the setup and then apply that knowledge to the rule stating that G must be advertised with either J or O. From this they are able to infer that J and O can never be advertised together: if J and O are advertised together, the pairing of J and O must occur in week 3, but then neither J nor O could be doubled, and one of J and O would have to be doubled in order to go with G. That line of reasoning indicates that answer choice (C) is correct.

For students who did not make that inference during their setup, there were still several steps available to help direct them to the correct answer. First, the correct answer in question #18 proves that H and K can be advertised during the same week, and so answer choice (A) can be eliminated with no work. Always remember to use the hypotheticals produced by earlier questions, especially List questions! Now, assume that none of the remaining answer choices appears noteworthy. The variables that cannot go together are probably prohibited from doing so because of their involvement with other variables. This makes it likely that any answer choice containing a random variable (a variable not cited in any rule) is less likely to be correct. L and M are the two randoms in the game, and answer choices (B), (D), and (E) each contain at least one random. Thus, answer choice (C) appears to be the most likely answer choice to initially analyze. This does not mean that you automatically choose answer choice (C) as the correct answer. It means that you can now closely examine (C) knowing that it has some characteristics that make it more likely to be correct than the other answer choices. Proceed by attempting to make a hypothetical with the two variables in (C). When a workable hypothetical cannot be produced, you will know that answer choice (C) is correct.

Get the most out of your LSAT Prep Plus subscription.

Analyze and track your performance with our Testing and Analytics Package.